Difference between revisions of "2005 AMC 12B Problems/Problem 6"
m |
(problem, solution, and category!) |
||
Line 1: | Line 1: | ||
− | {{ | + | \== Problem == |
− | == | + | In <math>\triangle ABC</math>, we have <math>AC=BC=7</math> and <math>AB=2</math>. Suppose that <math>D</math> is a point on line <math>AB</math> such that <math>B</math> lies between <math>A</math> and <math>D</math> and <math>CD=8</math>. What is <math>BD</math>? |
+ | == Solution == | ||
+ | Draw height <math>CH</math>. We have that <math>BH=1</math>. From the [[Pythagorean Theorem]], <math>CH=\sqrt{48}</math>. Since <math>CD=8</math>, <math>HD=\sqrt{8^2-48}=\sqrt{16}=4</math>, and <math>BD=HD-1</math>, so <math>BD=\boxed{3}</math>. | ||
− | |||
− | |||
== See also == | == See also == | ||
* [[2005 AMC 12B Problems/Problem 5 | Previous problem]] | * [[2005 AMC 12B Problems/Problem 5 | Previous problem]] | ||
* [[2005 AMC 12B Problems/Problem 7 | Next problem]] | * [[2005 AMC 12B Problems/Problem 7 | Next problem]] | ||
* [[2005 AMC 12B Problems]] | * [[2005 AMC 12B Problems]] | ||
+ | [[Category:Introductory Geometry Problems]] |
Revision as of 13:37, 4 January 2009
\== Problem == In , we have and . Suppose that is a point on line such that lies between and and . What is ?
Solution
Draw height . We have that . From the Pythagorean Theorem, . Since , , and , so .